Augmented matrix linear algebra

Click For Summary
The discussion focuses on determining conditions for the augmented matrix to have no solution, many solutions, or a unique solution based on the values of a and b. The key findings indicate that for no solution, b must equal 1/2 and a cannot equal 1; for many solutions, b must equal 1/2 and a must equal 1; and for a unique solution, b must not equal 1/2. Participants share their attempts at row reduction, with some achieving correct results while others seek clarification on their methods. Proper row reduction techniques are emphasized to arrive at the correct conditions for the system.
devilaudy
Messages
7
Reaction score
0

Homework Statement



Given the system whose augmented matrix is
 1 1 1 1 
 1 −1 0 a 
 0 1 b 0 
Determine (if possible) conditions on a and b such that this system has (a) no solution (b) many solutions (c) a unique solution.


Homework Equations



-Row reduction
-No solution: row of zero
-Many solutions: one or more free variables
-Unique solution: pivot in every column

The Attempt at a Solution



I tried to do it, but my answers were wrong. The good answers are a) b=1/2, a≠1 b) b=1/2, a=1 c) b≠1/2.

I tried to reduce the matrix and I had:

 0 0 1-2b 1-3a 
 1 0 b 2a 
 0 1 b a 

Can someone explain how to reduce the matrix properly or what am I doing wrong?
 
Physics news on Phys.org
devilaudy said:

Homework Statement



Given the system whose augmented matrix is
 1 1 1 1 
 1 −1 0 a 
 0 1 b 0 
Determine (if possible) conditions on a and b such that this system has (a) no solution (b) many solutions (c) a unique solution.


Homework Equations



-Row reduction
-No solution: row of zero
-Many solutions: one or more free variables
-Unique solution: pivot in every column

The Attempt at a Solution



I tried to do it, but my answers were wrong. The good answers are a) b=1/2, a≠1 b) b=1/2, a=1 c) b≠1/2.

I tried to reduce the matrix and I had:

 0 0 1-2b 1-3a 
 1 0 b 2a 
 0 1 b a 

Can someone explain how to reduce the matrix properly or what am I doing wrong?

Right off the bat I swapped the 2nd and 3rd rows. After that, I row reduced to get a matrix in echelon form. I get the same answers as your "good" ones.
 
Question: A clock's minute hand has length 4 and its hour hand has length 3. What is the distance between the tips at the moment when it is increasing most rapidly?(Putnam Exam Question) Answer: Making assumption that both the hands moves at constant angular velocities, the answer is ## \sqrt{7} .## But don't you think this assumption is somewhat doubtful and wrong?

Similar threads

  • · Replies 6 ·
Replies
6
Views
2K
  • · Replies 2 ·
Replies
2
Views
2K
  • · Replies 3 ·
Replies
3
Views
2K
Replies
4
Views
2K
  • · Replies 2 ·
Replies
2
Views
2K
Replies
2
Views
2K
  • · Replies 10 ·
Replies
10
Views
3K
  • · Replies 7 ·
Replies
7
Views
2K
  • · Replies 25 ·
Replies
25
Views
3K
  • · Replies 2 ·
Replies
2
Views
2K